Medsurg Exam 3: Chapters 39, 40, 38 sickle cell, transfusion
20. The nurse is administering a prescribed fibrinolytic to a client who is having a myocardial infarction (MI). Which adverse effect does the nurse monitor for? a.Bleedingb. Orthostatic hypotensionc.Deep vein thrombosis d.Nausea and vomiting
ANS: A A fibrinolytic lyses any clots in the body, thus causing an increased risk for bleeding. Fibrinolytic therapy does not place the client at risk for hypotension, thrombosis, or nausea and vomiting.
6. A client has Crohn's disease. What type of anemia is this client most at risk for developing? a.Folic acid deficiency b.Fanconi's anemia c.Hemolytic anemia d.Vitamin B12 anemia
ANS: A Malabsorption syndromes such as Crohn's disease leave a client prone to folic acid deficiency. Fanconi's anemia, hemolytic anemia, and vitamin B12 anemia are not related to Crohn's disease
4. A client presents to the emergency department in sickle cell crisis. What intervention by the nurse takes priority? a.Administer oxygen. b.Apply an oximetry probe. c.Give pain medication. d.Start an IV line.
ANS: A All actions are appropriate, but remembering the ABCs, oxygen would come first. The main problem in a sickle cell crisis is tissue and organ hypoxia, so providing oxygen helps halt the process
18. A nurse is caring for a client who is intubated and has an intra-aortic balloon pump. The client is restless and agitated. What action should the nurse perform first for comfort? a.Allow family members to remain at the bedside. b.Ask the family if the client would like a fan in the room. c.Keep the television tuned to the client's favorite channel. d.Speak loudly to the client in case of hearing problems.
ANS: A Allowing the family to remain at the bedside can help calm the client with familiar voices (and faces if the client wakes up). A fan might be helpful but may also spread germs through air movement. The TV should not be kept on all the time to allow for rest. Speaking loudly may agitate the client more
1. A nurse caring for a client with sickle cell disease (SCD) reviews the client's laboratory work. Which finding should the nurse report to the provider? a.Creatinine: 2.9 mg/dL b.Hematocrit: 30% c.Sodium: 147 mEq/L d. White blood cell count: 12,000/mm3
ANS: A An elevated creatinine indicates kidney damage, which occurs in SCD. A hematocrit level of 30% is an expected finding, as is a slightly elevated white blood cell count. A sodium level of 147 mEq/L, although slightly high, is not concerning.
8. A client is having a bone marrow biopsy and is extremely anxious. What action by the nurse is best? a.Assess client fears and coping mechanisms. b.Reassure the client this is a common test. c.Sedate the client prior to the procedure. d.Tell the client he or she will be asleep
ANS: A Assessing the client's specific fears and coping mechanisms helps guide the nurse in providing holistic care that best meets the client's needs. Reassurance will be helpful but is not the best option. Sedation is usually used. The client may or may not be totally asleep during the procedure.
3. A client in sickle cell crisis is dehydrated and in the emergency department. The nurse plans to start an IV. Which fluid choice is best? a.0.45% normal saline b.0.9% normal saline c.Dextrose 50% (D50) d.Lactated Ringer's solution
ANS: A Because clients in sickle cell crisis are often dehydrated, the fluid of choice is a hypotonic solution such as 0.45% normal saline. 0.9% normal saline and lactated Ringer's solution are isotonic. D50 is hypertonic and not used for hydration.
4. A nursing student is caring for a client who had a myocardial infarction. The student is confused because the client states nothing is wrong and yet listens attentively while the student provides education on lifestyle changes and healthy menu choices. What response by the faculty member is best? a."Continue to educate the client on possible healthy changes." b."Emphasize complications that can occur with noncompliance." c."Tell the client that denial is normal and will soon go away." d."You need to make sure the client understands this illness."
ANS: A Clients are often in denial after a coronary event. The client who seems to be in denial but is compliant with treatment may be using a healthy form of coping that allows time to process the event and start to use problem-focused coping. The student should not discourage this type of denial and coping, but rather continue providing education in a positive manner. Emphasizing complications may make the client defensive and more anxious. Telling the client that denial is normal is placing too much attention on the process. Forcing the client to verbalize understanding of the illness is also potentially threatening to the client.
3. The nurse is planning discharge teaching for a client who has acute myelogenous leukemia (AML). Which instruction does the nurse include in this client's discharge plan? a.Avoid contact sports.b. Refrain from intercourse.c.Apply heat to any bruised areas.d.Use aspirin for headaches.
ANS: A Clients with AML have a low platelet count and are at risk for bleeding. Contact sports can cause bleeding and should be avoided by those with a low platelet count. Anal intercourse should be avoided, but it is not necessary to refrain from all types of intercourse. Ice should be placed on bruised areas instead of heat, and aspirin should not be used by those with a low platelet count.
26. A client had an inferior wall myocardial infarction (MI). The nurse notes the client's cardiac rhythm as shown below: What action by the nurse is most important? a.Assess the client's blood pressure and level of consciousness. b.Call the health care provider or the Rapid Response Team. c.Obtain a permit for an emergency temporary pacemaker insertion. d. Prepare to administer antidysrhythmic medication.
ANS: A Clients with an inferior wall MI often have bradycardia and blocks that lead to decreased perfusion, as seen in this ECG strip showing sinus bradycardia. The nurse should first assess the client's hemodynamic status, including vital signs and level of consciousness. The client may or may not need the Rapid Response Team, a temporary pacemaker, or medication; there is no indication of this in the question.
2. A client hospitalized with sickle cell crisis frequently asks for opioid pain medications, often shortly after receiving a dose. The nurses on the unit believe the client is drug seeking. When the client requests pain medication, what action by the nurse is best? a.Give the client pain medication if it is time for another dose. b.Instruct the client not to request pain medication too early. c.Request the provider leave a prescription for a placebo. d.Tell the client it is too early to have more pain medication.
ANS: A Clients with sickle cell crisis often have severe pain that is managed with up to 48 hours of IV opioid analgesics. Even if the client is addicted and drug seeking, he or she is still in extreme pain. If the client can receive another dose of medication, the nurse should provide it. The other options are judgmental and do not address the client's pain. Giving placebos is unethical
3. The health care provider tells the nurse that a client is to be started on a platelet inhibitor. About what drug does the nurse plan to teach the client? a.Clopidogrel (Plavix) b.Enoxaparin (Lovenox) c.Reteplase (Retavase) d.Warfarin (Coumadin)
ANS: A Clopidogrel is a platelet inhibitor. Enoxaparin is an indirect thrombin inhibitor. Reteplase is a fibrinolytic agent. Warfarin is a vitamin K antagonist.
9. A client is in the clinic a month after having a myocardial infarction. The client reports sleeping well since moving into the guest bedroom. What response by the nurse is best? a."Do you have any concerns about sexuality?" b."I'm glad to hear you are sleeping well now." c."Sleep near your spouse in case of emergency." d."Why would you move into the guest room?"
ANS: A Concerns about resuming sexual activity are common after cardiac events. The nurse should gently inquire if this is the issue. While it is good that the client is sleeping well, the nurse should investigate the reason for the move. The other two responses are likely to cause the client to be defensive
21. A client is on a dopamine infusion via a peripheral line. What action by the nurse takes priority for safety? a.Assess the IV site hourly. b.Monitor the pedal pulses. c.Monitor the client's vital signs. d.Obtain consent for a central line.
ANS: A Dopamine should be infused through a central line to prevent extravasation and necrosis of tissue. If it needs to be run peripherally, the nurse assesses the site hourly for problems. When the client is getting the central line, ensuring informed consent is on the chart is a priority. But at this point, the client has only a peripheral line, so caution must be taken to preserve the integrity of the client's integumentary system. Monitoring pedal pulses and vital signs give indications as to how well the drug is working.
12. The nurse is caring for a client with leukemia who has the priority problem of fatigue. What action by the client best indicates that an important goal for this problem has been met? a.Doing activities of daily living (ADLs) using rest periods b.Helping plan a daily activity schedule c.Requesting a sleeping pill at night d.Telling visitors to leave when fatigued
ANS: A Fatigue is a common problem for clients with leukemia. This client is managing his or her own ADLs using rest periods, which indicates an understanding of fatigue and how to control it. Helping to plan an activity schedule is a lesser indicator. Requesting a sleeping pill does not help control fatigue during the day. Asking visitors to leave when tired is another lesser indicator. Managing ADLs using rest periods demonstrates the most comprehensive management strategy.
11. An older adult is on cardiac monitoring after a myocardial infarction. The client shows frequent dysrhythmias. What action by the nurse is most appropriate? a.Assess for any hemodynamic effects of the rhythm. b.Prepare to administer antidysrhythmic medication. c.Notify the provider or call the Rapid Response Team. d.Turn the alarms off on the cardiac monitor.
ANS: A Older clients may have dysrhythmias due to age-related changes in the cardiac conduction system. They may have no significant hemodynamic effects from these changes. The nurse should first assess for the effects of the dysrhythmia before proceeding further. The alarms on a cardiac monitor should never be shut off. The other two actions may or may not be needed.
2. A client is receiving rivaroxaban (Xarelto) and asks the nurse to explain how it works. What response by the nurse is best? a. "It inhibits thrombin." b. "It inhibits fibrinogen." c. "It thins your blood." d. "It works against vitamin K."
ANS: A Rivaroxaban is a direct thrombin inhibitor. It does not work on fibrinogen or vitamin K. It is not a "blood thinner," although many clients call anticoagulants by this name.
25. A nurse is caring for four clients with leukemia. After hand-off report, which client should the nurse see first? a.Client who had two bloody diarrhea stools this morning b.Client who has been premedicated for nausea prior to chemotherapy c.Client with a respiratory rate change from 18 to 22 breaths/min d.Client with an unchanged lesion to the lower right lateral malleolus
ANS: A The client who had two bloody diarrhea stools that morning may be hemorrhaging in the gastrointestinal (GI) tract and should be assessed first. The client with the change in respiratory rate may have an infection or worsening anemia and should be seen next. The other two clients are not a priority at this time.
14. A client has been admitted after sustaining a humerus fracture that occurred when picking up the family cat. What test result would the nurse correlate to this condition? a.Bence-Jones protein in urine b.Epstein-Barr virus: positive c.Hemoglobin: 18 mg/dL d. Red blood cell count: 8.2/mm3
ANS: A This client has possible multiple myeloma. A positive Bence-Jones protein finding would correlate with this condition. The Epstein-Barr virus is a herpesvirus that causes infectious mononucleosis and some cancers. A hemoglobin of 18 mg/dL is slightly high for a male and somewhat high for a female; this can be caused by several conditions, and further information would be needed to correlate this value with a specific medical condition. A red blood cell count of 8.2/mm3 is also high, but again, more information would be needed to correlate this finding with a specific medical condition.
4. A nurse is assessing a dark-skinned client for pallor. What action is best? a.Assess the conjunctiva of the eye. b.Have the client open the hand widely. c.Look at the roof of the client's mouth. d.Palpate for areas of mild swelling.
ANS: A To assess pallor in dark-skinned people, assess the conjunctiva of the eye or the mucous membranes. Looking at the roof of the mouth can reveal jaundice. Opening the hand widely is not related to pallor, nor is palpating for mild swelling.
17. The nurse is assessing a client whose warfarin (Coumadin) therapy was discontinued 3 weeks ago. Which laboratory test result indicates that the client's warfarin therapy is no longer therapeutic? a.International normalized ratio (INR), 0.9b. Reticulocyte count, 1%c.Serum ferritin level, 350 ng/mLd.Total white blood cell (WBC) count, 9000/mm
ANS: A Warfarin therapy increases the INR. Normal INR ranges between 0.7 and 1.8. Therapeutic warfarin levels, depending on the indication of the disorder, should maintain the INR between 1.5 and 3.0. When the effects of warfarin are no longer present, the INR returns to normal levels. Warfarin therapy does not affect white blood cell count, serum ferritin level, or reticulocyte count.
26. A client has frequent hospitalizations for leukemia and is worried about functioning as a parent to four small children. What action by the nurse would be most helpful? a.Assist the client to make "sick day" plans for household responsibilities. b.Determine if there are family members or friends who can help the client. c.Help the client inform friends and family that they will have to help out. d.Refer the client to a social worker in order to investigate respite child care.
ANS: A While all options are reasonable choices, the best option is to help the client make sick day plans, as that is more comprehensive and inclusive than the other options, which focus on a single item.
17. A client has a platelet count of 9000/mm3. The nurse finds the client confused and mumbling. What action takes priority? a.Calling the Rapid Response Team b.Delegating taking a set of vital signs c.Instituting bleeding precautions d.Placing the client on bedrest
ANS: A With a platelet count this low, the client is at high risk of spontaneous bleeding. The most disastrous complication would be intracranial bleeding. The nurse needs to call the Rapid Response Team as this client has manifestations of a sudden neurologic change. The nurse should not delegate the vital signs as the client is no longer stable. Bleeding precautions will not address the immediate situation. Placing the client on bedrest or putting the client back into bed is important, but the critical action is to call for immediate medical attention.
10. The nurse is obtaining the health history of a client who has iron deficiency anemia. Which factor in this client's history does the nurse correlate with this diagnosis? a.Eating a meat-free dietb. Family history of sickle cell diseasec.History of leukemiad.History of bleeding ulcer
ANS: A A diet high in protein and iron helps keep the client's levels of iron within normal limits. Meat is a good source of protein and iron. A bleeding ulcer could cause anemia but would not cause iron deficiency. Sickle cell disease causes sickle cell anemia. Leukemia causes a decrease in white blood cells.
21. The nurse is assessing a client who has undergone a percutaneous transluminal coronary angioplasty (PTCA) and is ordered to receive an IV infusion of abciximab (ReoPro). Which clinical manifestation does the nurse monitor for in this client? a.Bleedingb. Joint painc.Pedal edemad.Excessive thirst
ANS: A Administration of glycoprotein (GP) IIa/IIIb inhibitors is common during the first few hours after PTCA. The nurse should monitor the client closely for bleeding and hypersensitivity reactions, which can include angioedema, urticaria, and even anaphylaxis. The other manifestations are not associated with the administration of GP IIa/IIIb inhibitors.
21. The nurse is caring for a client who had a bone marrow aspiration. The client begins to bleed from the aspiration site. Which action does the nurse perform? a.Apply external pressure to the site.b. Elevate the extremities.c.Cover the site with a dressing.d.Immobilize the leg.
ANS: A All these options could be done after a bone marrow aspiration and biopsy. However, the most important action when bleeding occurs is to apply external pressure to the site until hemostasis is ensured. The other measures could then be carried out.
7. The nurse evaluates diagnostic results for a client who has chest pain. Which laboratory test is most specific for acute coronary syndromes? a.Troponin markersb. Serum lactate dehydrogenase (LDH)c.Serum myoglobind.Creatine kinase (CK)-MB isoenzyme
ANS: A Although all these laboratory tests are appropriate to confirm or rule out a myocardial infarction, the one most specific for acute coronary syndromes is troponin T. When elevated, it serves to identify the development of unstable angina, subendocardial MI, or MI.
19. The nurse is assessing a client with anemia. Which clinical manifestation does the nurse expect to see in this client? a.Dyspnea with activityb. Hypertensionc.Bradycardiad.Warm, flushed skin
ANS: A Anemia is a reduction in the number of red blood cells (RBCs), the amount of hemoglobin, or the hematocrit level. Tissue oxygenation depends on RBCs. Typical symptoms of anemic clients include dyspnea, increased somnolence, tachycardia, and pallor. A client who is anemic tends to have lower blood pressure, increased heart rate, and skin that is pale and cool to touch
17. The nurse is teaching a client who is prescribed a calcium channel blocking agent after a percutaneous transluminal coronary angioplasty (PTCA). Which instruction does the nurse include in this client's teaching? a." Change position slowly." b. " Avoid crossing your legs." c." Weigh yourself daily." d." Decrease salt intake."
ANS: A Calcium channel blocking agents cause systemic vasodilation and postural (orthostatic) hypotension. The client should avoid crossing legs, should weigh daily, and should decrease salt intake, but these are not associated with teaching for a calcium channel blocker.
11. The nurse is caring for a client who has a decreased serum iron level. Which intervention does the nurse prioritize for this client? a.Dietary consultb. Family assessmentc.Cardiac assessmentd.Administration of vitamin K
ANS: A Diets can alter cell quality and affect blood clotting. Diets low in iron can cause anemia and decrease the function of all red blood cells. The question does not say that the hemoglobin is low enough to affect the cardiac function. Family assessment may be important in finding out any genetic or family lifestyle causes of the low serum iron level. However, the first intervention that the nurse can provide is to have the client's dietary habits evaluated and changed so that iron levels can increase. Vitamin K is involved with clotting, not with iron stores.
8. The nurse is assessing a client who has a factor VIII deficiency. Which clinical manifestation does the nurse expect to assess in this client? a.Excessive bleeding from a cutb. Chronic lower back painc.Nausea and vomitingd.Temperature of 101° F
ANS: A Factor VIII deficiency is also known as hemophilia A. With hemophilia, a client has a prolonged partial thromboplastin time (PTT) and is at risk for excessive bleeding from minor cuts. The other three distractors are not associated with a factor VIII deficiency.
12. The nurse is administering thrombolytic therapy to a client who had a myocardial infarction. Which intervention does the nurse implement to reduce the risk of complications in this client? a.Administer prescribed heparin.b. Apply ice to the injection site. c.Place the client in Trendelenburg position.d.Instruct the client to take slow deep breaths
ANS: A Following clot lysis, large amounts of thrombin are released, increasing the risk of vessel reocclusion. To maintain vessel patency, IV or low-molecular-weight heparin and aspirin are prescribed. The other interventions are not appropriate for this client.
8. The nurse is assessing a 75-year-old male client. Which blood value indicates that the client is experiencing normal changes associated with aging? a.Hemoglobin, 13.0 g/dLb. Platelet count, 100,000/mm3c.Prothrombin time (PT), 14 secondsd.White blood cell (WBC) count, 5000/mm3
ANS: A Hemoglobin levels in men and women fall after middle age. Therefore, this client's hemoglobin value would be considered part of the aging process. Platelet counts and blood-clotting times are not age related; the client's platelet count and PT are elevated for some other reason. The WBC count shown is normal.
4. The nurse is assessing a client with numerous areas of bruising. Which question does the nurse ask to determine the cause of this finding? a."Do you take aspirin?"b. "How often do you exercise?"c."Are you a vegetarian?"d."How often do you take Tylenol?"
ANS: A Platelet aggregation is essential for blood clotting. An inability to clot blood when an injury occurs can result in bleeding, which would cause bruising. Aspirin is a drug that interferes with platelet aggregation and has the ability to "plug" an extrinsic event, such as trauma. Vitamin K found in green vegetables enhances clotting factors, which would improve the ability to stop bleeding associated with an extrinsic event. Acetaminophen (Tylenol) and exercise do not inhibit clotting factors.
15. The nurse is preparing a client for surgery. The client states, " I am concerned I might be given blood products during surgery and this would be against my religious beliefs." How does the nurse respond? a." We can use other means to replace blood loss besides blood products." b. " Your chance of needing a blood transfusion is small." c." The operating team will do what is necessary to save your life." d." You could have family members donate blood for you."
ANS: A The client's rights and wishes should be respected while accurate information is provided for reassurance. Directed donations from family members neither ensure safe blood products nor may be sanctioned by the client's religion.
16. The nurse is caring for a client who had a myocardial infarction. The client develops increased pulmonary congestion; an increase in heart rate from 80 to 102 beats/min; and cold, clammy skin. Which action does the nurse implement before notifying the health care provider? a.Administer oxygen.b. Increase the IV flow rate.c.Place the client in supine position.d.Prepare the client for surgery.
ANS: A The nurse recognizes these manifestations as impending cardiogenic shock. Oxygen is needed to prevent further deterioration. The provider is notified immediately so that efforts can be made to reverse this condition because it has a mortality rate of 65% to 100%. IV fluids would enhance the respiratory edema. The client should be placed in high Fowler's position to assist with respirations. The client does not need surgery.
19. The nurse is teaching a client prescribed sublingual nitroglycerin for chest pain. Which statement indicates that the client needs further teaching? a." I carry my medicine around in a clear plastic bag so that I can get to it easily if I have chest pain." b. " Even if I have not used any of the nitroglycerin from one refill, I get another refill every 3 months." c." If I still have chest pain after I have taken 3 nitroglycerin tablets, I will go to the hospital." d." When my nitroglycerin tablet tingles under my tongue, I know that it is strong enough to work."
ANS: A The shelf life of nitroglycerin is short. It deteriorates quickly in the presence of light or moisture. A clear plastic bag does not provide sufficient protection to ensure potency of the drug. Nitroglycerin tablets should be replaced every 3 to 5 months. If chest pain continues after taking nitroglycerin, the client should call EMS. Nitroglycerin is given sublingual.
6. The nurse is planning discharge teaching for a client who has a splenectomy. Which statement does the nurse include in this client's teaching plan? a."Avoid crowds and people who are sick."b. "Do not eat raw fruits or vegetables."c."Avoid environmental allergens."d."Do not play contact sports."
ANS: A The spleen is the major site of B-lymphocyte maturation and antibody production. Those who undergo splenectomies for any reason have a decreased antibody-mediated immune response and are particularly susceptible to viral infections. Eating raw fruits and vegetables places the client at risk for bacterial infections. The body responds to environmental allergens with an unspecific inflammatory process. The client is not at risk for bleeding or injury due to contact sports.
4. The nurse is caring for a client who has autoimmune thrombocytopenic purpura. Which intervention does the nurse implement for this client? a.Avoid intramuscular injections.b. Administer prescribed anticoagulants.c.Infuse intravenous normal saline.d.Monitor for an increase in temperature.
ANS: A With autoimmune thrombocytopenic purpura, the total number of circulating platelets is greatly reduced. As a result of the decreased platelet count, the client is at great risk for bleeding, and intramuscular injections should be avoided. Anticoagulants should not be given. A low platelet count is not treated with saline, and thrombocytopenia will not cause a change in body temperature
8. A nurse is preparing to administer a blood transfusion to an older adult. Understanding age-related changes, what alterations in the usual protocol are necessary for the nurse to implement? (Select all that apply.) a.Assess vital signs more often. b.Hold other IV fluids running. c.Premedicate to prevent reactions. d.Transfuse smaller bags of blood. e.Transfuse each unit over 8 hours.
ANS: A, B The older adult needs vital signs monitored as often as every 15 minutes for the duration of the transfusion because changes may be the only indication of a transfusion-related problem. To prevent fluid overload, the nurse obtains a prescription to hold other running IV fluids during the transfusion. The other options are not warranted.
5. A nursing student planning to teach clients about risk factors for coronary artery disease (CAD) would include which topics? (Select all that apply.) a.Advanced age b.Diabetes c.Ethnic background d.Medication use e.Smoking
ANS: A, B, C, E Age, diabetes, ethnic background, and smoking are all risk factors for developing CAD; medication use is not
5. A student nurse is helping a registered nurse with a blood transfusion. Which actions by the student are most appropriate? (Select all that apply.) a.Hanging the blood product using normal saline and a filtered tubing set b.Taking a full set of vital signs prior to starting the blood transfusion c.Telling the client someone will remain at the bedside for the first 5 minutes d.Using gloves to start the client's IV if needed and to handle the blood product e.Verifying the client's identity, and checking blood compatibility and expiration time
ANS: A, B, D Correct actions prior to beginning a blood transfusion include hanging the product with saline and the correct filtered blood tubing, taking a full set of vital signs prior to starting, and using gloves. Someone stays with the client for the first 15 to 30 minutes of the transfusion. Two registered nurses must verify the client's identity and blood compatibility.
9. A client has heparin-induced thrombocytopenia (HIT). The student nurse asks how this is treated. About what drugs does the nurse instructor teach? (Select all that apply.) a.Argatroban (Argatroban) b.Bivalirudin (Angiomax) c.Clopidogrel (Plavix) d.Lepirudin (Refludan) e.Methylprednisolone (Solu-Medrol)
ANS: A, B, D The standard drugs used to treat HIT are argatroban, bivalirudin, and lepirudin. The other drugs are not used. Clopidogrel is an antiplatelet agent used to reduce the likelihood of stroke or myocardial infarction. Methylprednisolone is a steroid used to reduce inflammation.
3. A nursing student studying acute coronary syndromes learns that the pain of a myocardial infarction (MI) differs from stable angina in what ways? (Select all that apply.) a.Accompanied by shortness of breath b.Feelings of fear or anxiety c.Lasts less than 15 minutes d.No relief from taking nitroglycerin e.Pain occurs without known cause
ANS: A, B, D, E The pain from an MI is often accompanied by shortness of breath and fear or anxiety. It lasts longer than 15 minutes and is not relieved by nitroglycerin. It occurs without a known cause such as exertion.
15. A client with coronary artery disease (CAD) asks the nurse about taking fish oil supplements. What response by the nurse is best? a."Fish oil is contraindicated with most drugs for CAD." b."The best source is fish, but pills have benefits too." c."There is no evidence to support fish oil use with CAD." d."You can reverse CAD totally with diet and supplements."
ANS: B Omega-3 fatty acids have shown benefit in reducing lipid levels, in reducing the incidence of sudden cardiac death, and for stabilizing atherosclerotic plaque. The best source is fish three times a week or some fish oil supplements. The other options are not accurate.
2. The nurse is teaching a client who is receiving sodium warfarin (Coumadin). Which topics does the nurse include in the teaching plan? (Select all that apply.) a.Foods high in vitamin Kb. Using acetaminophen (Tylenol) for minor painc.Daily exercise and weight managementd.Use of a safety razor and soft toothbrushe.Blood testing regimen
ANS: A, B, D, E The client on warfarin will need to know which foods are high in vitamin K because vitamin K intake must be consistent to avoid interfering with the anticoagulant properties of warfarin. Clients should not take aspirin or NSAIDs for minor pain owing to their anticoagulant properties. Clients must use safety razors and soft toothbrushes to avoid bleeding episodes. The client on warfarin needs regular blood tests for prothrombin time (PT) and international normalized ratio (INR). Daily exercise and weight management are not specifically important to this client
1. A student nurse learns that the spleen has several functions. What functions do they include? (Select all that apply.) a.Breaks down hemoglobin b.Destroys old or defective red blood cells (RBCs) c.Forms vitamin K for clotting d.Stores extra iron in ferritin e.Stores platelets not circulating
ANS: A, B, E Functions of the spleen include breaking down hemoglobin released from RBCs, destroying old or defective RBCs, and storing the platelets that are not in circulation. Forming vitamin K for clotting and storing extra iron in ferritin are functions of the liver.
2. An older client asks the nurse why "people my age" have weaker immune systems than younger people. What responses by the nurse are best? (Select all that apply.) a. "Bone marrow produces fewer blood cells." b. "You may have decreased levels of circulating platelets." c. "You have lower levels of plasma proteins in the blood." d. "Lymphocytes become more reactive to antigens." e. "Spleen function declines after age 60."
ANS: A, C The aging adult has bone marrow that produces fewer cells and decreased blood volume with fewer plasma proteins. Platelet numbers remain unchanged, lymphocytes become less reactive, and spleen function stays the same.
1. The nurse is monitoring a client with liver failure. Which assessments does the nurse perform when monitoring for bleeding in this client? (Select all that apply.) a.Gumsb. Lung soundsc.Urined.Stoole.Hair
ANS: A, C, D The liver is the site for production of clotting factors. Without these factors, the client is at risk for bleeding. Common areas of bleeding include the gums and mucous membranes, bladder, and gastrointestinal tract. Lung sounds and hair are part of the assessment but are not essential in the presence of liver failure and hematologic abnormalities
1. A nurse working with clients with sickle cell disease (SCD) teaches about self-management to prevent exacerbations and sickle cell crises. What factors should clients be taught to avoid? (Select all that apply.) a.Dehydration b.Exercise c.Extreme stress d. High altitudes e.Pregnancy
ANS: A, C, D, E Several factors cause red blood cells to sickle in SCD, including dehydration, extreme stress, high altitudes, and pregnancy. Strenuous exercise can also cause sickling, but not unless it is very vigorous.
2. A student studying leukemias learns the risk factors for developing this disorder. Which risk factors does this include? (Select all that apply.) a.Chemical exposure b.Genetically modified foods c.Ionizing radiation exposure d.Vaccinations e.Viral infections
ANS: A, C, E Chemical and ionizing radiation exposure and viral infections are known risk factors for developing leukemia. Eating genetically modified food and receiving vaccinations are not known risk factors.
2. A nurse is caring for a client who had coronary artery bypass grafting yesterday. What actions does the nurse delegate to the unlicensed assistive personnel (UAP)? (Select all that apply.) a.Assist the client to the chair for meals and to the bathroom. b.Encourage the client to use the spirometer every 4 hours. c.Ensure the client wears TED hose or sequential compression devices. d.Have the client rate pain on a 0-to-10 scale and report to the nurse. e.Take and record a full set of vital signs per hospital protocol.
ANS: A, C, E The nurse can delegate assisting the client to get up in the chair or ambulate to the bathroom, applying TEDs or sequential compression devices, and taking/recording vital signs. The spirometer should be used every hour the day after surgery. Assessing pain using a 0-to-10 scale is a nursing assessment, although if the client reports pain, the UAP should inform the nurse so a more detailed assessment is done.
6. A student nurse is learning about blood transfusion compatibilities. What information does this include? (Select all that apply.) a.Donor blood type A can donate to recipient blood type AB. b.Donor blood type B can donate to recipient blood type O. c.Donor blood type AB can donate to anyone. d.Donor blood type O can donate to anyone. e.Donor blood type A can donate to recipient blood type B.
ANS: A, D Blood type A can be donated to people who have blood types A or AB. Blood type O can be given to anyone. Blood type B can be donated to people who have blood types B or AB. Blood type AB can only go to recipients with blood type AB
4. A client is hospitalized after a myocardial infarction. Which hemodynamic parameters does the nurse correlate with cardiogenic shock? (Select all that apply.) a.Decreased cardiac outputb. Increased cardiac outputc.Increased mean arterial pressure (MAP)d.Decreased MAPe.Increased afterloadf.Decreased afterload
ANS: A, D, E Myocardial infarction (MI) is a major cause of direct pump failure. With MI, cardiac output and MAP are decreased and afterload is increased. The other parameters do not correlate with pump failure.
1. The nurse is planning a community health promotion program for cardiovascular disease. Which risk factors of coronary artery disease (CAD) does the nurse include in the education? (Select all that apply.) a.Cigarette smokingb. Use of alcoholc.Insomniad.Hypertensione.Obesityf.Depression
ANS: A, D, E Teach about lifestyle risk factors of CAD, such as obesity, smoking, positive family history, cholesterol management, and diagnosis and treatment of hypertension
6. A client is having a bone marrow biopsy today. What action by the nurse takes priority? a.Administer pain medication first. b.Ensure valid consent is on the chart. c.Have the client shower in the morning. d.Premedicate the client with sedatives.
ANS: B A bone marrow biopsy is an invasive procedure that requires informed consent. Pain medication and sedation are important components of care for this client but do not take priority. The client may or may not need or be able to shower.
7. A client has intra-arterial blood pressure monitoring after a myocardial infarction. The nurse notes the client's heart rate has increased from 88 to 110 beats/min, and the blood pressure dropped from 120/82 to 100/60 mm Hg. What action by the nurse is most appropriate? a.Allow the client to rest quietly. b.Assess the client for bleeding. c.Document the findings in the chart. d.Medicate the client for pain.
ANS: B A major complication related to intra-arterial blood pressure monitoring is hemorrhage from the insertion site. Since these vital signs are out of the normal range, are a change, and are consistent with blood loss, the nurse should assess the client for any bleeding associated with the arterial line. The nurse should document the findings after a full assessment. The client may or may not need pain medication and rest; the nurse first needs to rule out any emergent bleeding
2. A client received tissue plasminogen activator (t-PA) after a myocardial infarction and now is on an intravenous infusion of heparin. The client's spouse asks why the client needs this medication. What response by the nurse is best? a."The t-PA didn't dissolve the entire coronary clot." b."The heparin keeps that artery from getting blocked again." c."Heparin keeps the blood as thin as possible for a longer time." d."The heparin prevents a stroke from occurring as the t-PA wears off."
ANS: B After the original intracoronary clot has dissolved, large amounts of thrombin are released into the bloodstream, increasing the chance of the vessel reoccluding. The other statements are not accurate. Heparin is not a "blood thinner," although laypeople may refer to it as such.
7. A nurse in a hematology clinic is working with four clients who have polycythemia vera. Which client should the nurse see first? a.Client with a blood pressure of 180/98 mm Hg b.Client who reports shortness of breath c.Client who reports calf tenderness and swelling d.Client with a swollen and painful left great toe
ANS: B Clients with polycythemia vera often have clotting abnormalities due to the hyperviscous blood with sluggish flow. The client reporting shortness of breath may have a pulmonary embolism and should be seen first. The client with a swollen calf may have a deep vein thrombosis and should be seen next. High blood pressure and gout symptoms are common findings with this disorder.
20. A home health care nurse is visiting an older client who lives alone after being discharged from the hospital after a coronary artery bypass graft. What finding in the home most causes the nurse to consider additional referrals? a.Dirty carpets in need of vacuuming b.Expired food in the refrigerator c.Old medications in the kitchen d.Several cats present in the home
ANS: B Expired food in the refrigerator demonstrates a safety concern for the client and a possible lack of money to buy food. The nurse can consider a referral to Meals on Wheels or another home-based food program. Dirty carpets may indicate the client has no household help and is waiting for clearance to vacuum. Old medications can be managed by the home health care nurse and the client working collaboratively. Having pets is not a cause for concern.
14. A nurse is in charge of the coronary intensive care unit. Which client should the nurse see first? a.Client on a nitroglycerin infusion at 5 mcg/min, not titrated in the last 4 hours b.Client who is 1 day post coronary artery bypass graft, blood pressure 180/100 mm Hg c.Client who is 1 day post percutaneous coronary intervention, going home this morning d.Client who is 2 days post coronary artery bypass graft, became dizzy this a.m. while walking
ANS: B Hypertension after coronary artery bypass graft surgery can be dangerous because it puts too much pressure on the suture lines and can cause bleeding. The charge nurse should see this client first. The client who became dizzy earlier should be seen next. The client on the nitroglycerin drip is stable. The client going home can wait until the other clients are cared for
6. A client has hemodynamic monitoring after a myocardial infarction. What safety precaution does the nurse implement for this client? a.Document pulmonary artery wedge pressure (PAWP) readings and assess their trends. b.Ensure the balloon does not remain wedged. c.Keep the client on strict NPO status. d.Maintain the client in a semi-Fowler's position.
ANS: B If the balloon remains inflated, it can cause pulmonary infarction or rupture. The nurse should ensure the balloon remains deflated between PAWP readings. Documenting PAWP readings and assessing trends is an important nursing action related to hemodynamic monitoring, but is not specifically related to safety. The client does not have to be NPO while undergoing hemodynamic monitoring. Positioning may or may not affect readings.
19. The nurse is caring for a client with a chest tube after a coronary artery bypass graft. The drainage slows significantly. What action by the nurse is most important? a.Increase the setting on the suction. b.Notify the provider immediately. c.Re-position the chest tube. d.Take the tubing apart to assess for clots.
ANS: B If the drainage in the chest tube decreases significantly and dramatically, the tube may be blocked by a clot. This could lead to cardiac tamponade. The nurse should notify the provider immediately. The nurse should not independently increase the suction, re-position the chest tube, or take the tubing apart
18. A nurse is preparing to administer a blood transfusion. What action is most important? a.Correctly identifying client using two identifiers b.Ensuring informed consent is obtained if required c.Hanging the blood product with Ringer's lactate d. Staying with the client for the entire transfusion
ANS: B If the facility requires informed consent for transfusions, this action is most important because it precedes the other actions taken during the transfusion. Correctly identifying the client and blood product is a National Patient Safety Goal, and is the most important action after obtaining informed consent. Ringer's lactate is not used to transfuse blood. The nurse does not need to stay with the client for the duration of the transfusion.
9. The family of a neutropenic client reports the client "is not acting right." What action by the nurse is the priority? a.Ask the client about pain. b.Assess the client for infection. c.Delegate taking a set of vital signs. d.Look at today's laboratory results.
ANS: B Neutropenic clients often do not have classic manifestations of infection, but infection is the most common cause of death in neutropenic clients. The nurse should assess for infection. The nurse should assess for pain but this is not the priority. The nurse should take the client's vital signs instead of delegating them since the client has had a change in status. Laboratory results may be inconclusive.
16. A client has presented to the emergency department with an acute myocardial infarction (MI). What action by the nurse is best to meet The Joint Commission's Core Measures outcomes? a.Obtain an electrocardiogram (ECG) now and in the morning. b.Give the client an aspirin. c.Notify the Rapid Response Team. d.Prepare to administer thrombolytics.
ANS: B The Joint Commission's Core Measures set for acute MI require that aspirin is administered when a client with MI presents to the emergency department or when an MI occurs in the hospital. A rapid ECG is vital, but getting another one in the morning is not part of the Core Measures set. The Rapid Response Team is not needed if an emergency department provider is available. Thrombolytics may or may not be needed
23. A client admitted for sickle cell crisis is distraught after learning her child also has the disease. What response by the nurse is best? a."Both you and the father are equally responsible for passing it on." b."I can see you are upset. I can stay here with you a while if you like." c."It's not your fault; there is no way to know who will have this disease." d."There are many good treatments for sickle cell disease these days."
ANS: B The best response is for the nurse to offer self, a therapeutic communication technique that uses presence. Attempting to assign blame to both parents will not help the client feel better. There is genetic testing available, so it is inaccurate to state there is no way to know who will have the disease. Stating that good treatments exist belittles the client's feelings.
1. The nurse is taking the history of a client with suspected coronary artery disease (CAD). Which situation correlates with stable angina? a.Chest discomfort at rest and inability to tolerate mowing the lawn b. Chest discomfort when mowing the lawn and subsiding with rest c.Indigestion and a choking sensation when mowing the lawn d.Jaw pain that radiates to the shoulder after mowing the lawn
ANS: B The client with stable angina reports chest discomfort that occurs with moderate, prolonged exertion. This discomfort is typically relieved with nitroglycerin or rest. The other experiences do not correlate with stable angina.
17. A nurse is caring for four clients. Which client should the nurse assess first? a.Client with an acute myocardial infarction, pulse 102 beats/min b.Client who is 1 hour post angioplasty, has tongue swelling and anxiety c.Client who is post coronary artery bypass, chest tube drained 100 mL/hr d.Client who is post coronary artery bypass, potassium 4.2 mEq/L
ANS: B The post-angioplasty client with tongue swelling and anxiety is exhibiting manifestations of an allergic reaction that could progress to anaphylaxis. The nurse should assess this client first. The client with a heart rate of 102 beats/min may have increased oxygen demands but is just over the normal limit for heart rate. The two post coronary artery bypass clients are stable.
20. A client receiving a blood transfusion develops anxiety and low back pain. After stopping the transfusion, what action by the nurse is most important? a.Documenting the events in the client's medical record b.Double-checking the client and blood product identification c.Placing the client on strict bedrest until the pain subsides d.Reviewing the client's medical record for known allergies
ANS: B This client had a hemolytic transfusion reaction, most commonly caused by blood type or Rh incompatibility. The nurse should double-check all identifying information for both the client and blood type. Documentation occurs after the client is stable. Bedrest may or may not be needed. Allergies to medications or environmental items is not related
5. A client has a serum ferritin level of 8 ng/mL and microcytic red blood cells. What action by the nurse is best? a.Encourage high-protein foods. b.Perform a Hemoccult test on the client's stools. c.Offer frequent oral care. d.Prepare to administer cobalamin (vitamin B12).
ANS: B This client has laboratory findings indicative of iron deficiency anemia. The most common cause of this disorder is blood loss, often from the GI tract. The nurse should perform a Hemoccult test on the client's stools. High-protein foods may help the condition, but dietary interventions take time to work. That still does not determine the cause. Frequent oral care is not related. Cobalamin injections are for pernicious anemia
3. A client is in the hospital after suffering a myocardial infarction and has bathroom privileges. The nurse assists the client to the bathroom and notes the client's O2 saturation to be 95%, pulse 88 beats/min, and respiratory rate 16 breaths/min after returning to bed. What action by the nurse is best? a.Administer oxygen at 2 L/min. b.Allow continued bathroom privileges. c.Obtain a bedside commode. d.Suggest the client use a bedpan.
ANS: B This client's physiologic parameters did not exceed normal during and after activity, so it is safe for the client to continue using the bathroom. There is no indication that the client needs oxygen, a commode, or a bedpan
8. While evaluating a client's electrocardiogram (ECG) before surgery, the preoperative nurse identifies large, wide Q waves. What is the nurse's best interpretation of this finding? a.An acute myocardial infarction is occurring.b. The client had a myocardial infarction in the past.c.The ventricles are enlarged and failing.d.The ECG is a common variation of normal sinus rhythm.
ANS: B A wide and large Q wave develops as a result of myocardial infarction and necrotic ventricular cells that do not conduct electrical impulses. This change is usually permanent. When it appears alone, it indicates a past MI. The other interpretations are not correct.
The nurse is assessing a client who has a history of stable angina. The client describes a recent increase in the number of attacks and in the intensity of the pain. Which question does the nurse ask to assess the client's change in condition? a." How many cigarettes do you smoke daily?" b. " Do you have pain when you are resting?" c." Do you have abdominal pain or nausea?" d." How frequently are you having chest pain?"
ANS: B An increase in the number of anginal attacks and an increase in the intensity of pain characterize unstable angina. Chest pain or discomfort also occurs at rest. The nurse should assess for this characteristic of unstable angina. The other questions would not be helpful in assessing for unstable angina.
7. The nurse is teaching a client who has undergone a bone marrow biopsy. Which instruction does the nurse give the client? a."Wear protective gear when playing contact sports."b. "Monitor the biopsy site for bruising."c."Remain in bed for at least 12 hours."d."Use a heating pad for pain at the biopsy site."
ANS: B The most important instruction is to have the client monitor the area for external or internal bleeding. Activities such as contact sports should be avoided, and an ice pack can be used to limit bruising
18. The nurse is completing the preoperative checklist on a client. The client states, "I take an aspirin every day for my heart." How does the nurse respond? a."I will call your doctor and request a prescription for pain medication."b. "I need to call the surgeon and reschedule your surgery."c."I'll give you the prescribed Tylenol to minimize any headache before surgery."d."I need to administer vitamin K to prevent bleeding during the procedure."
ANS: B Aspirin and other salicylates interfere with platelet aggregation—the first step in the blood-clotting cascade—and decrease the ability of the blood to form a platelet plug. These effects last for longer than 1 week after just one dose of aspirin. The client may need to have the surgery rescheduled. Vitamin K, prescribed pain medication, and Tylenol cannot reduce the anticlotting effects of aspirin
14. The nurse is assisting a client to walk in the hall on the third day after a myocardial infarction. Which clinical manifestation indicates to the nurse that the client is not ready to advance to the next level of activity? a.Facial flushingb. Onset of chest painc.Heart rate increase of 10 beats/min at completion of the activityd.Systolic blood pressure increase of 10 mm Hg at completion of the activity
ANS: B Chest pain on ambulation indicates poor tolerance to activity and is an indication that the heart is not ready for progression. The other manifestations indicate that the client is tolerating the activity
13. Which risk factor does the nurse assess for to determine a client's cause of anemia? a.Antacid therapyb. Chronic alcoholismc.Congestive heart failured.Type 2 diabetes
ANS: B Chronic alcohol abuse is strongly associated with malnutrition of many dietary essentials, including iron, folic acid, and vitamin B12. Antacids, heart failure, and diabetes affect nutrition at varying levels, but anemia is most closely related to the malnutrition seen with chronic alcohol abuse
25. The nurse observes that a client, whose blood type is AB-negative, is receiving a transfusion with type O-negative packed red blood cells. Which action does the nurse take first? a.Report the problem to the blood bank.b. Assess and record the client's vital signs.c.Stop the transfusion and keep the IV open.d.Administer prescribed diphenhydramine (Benadryl)
ANS: B Clients with an AB-negative blood type can receive O-negative blood because they do not have antibodies against this type of blood. The transfusion can proceed. The nurse monitors the client's vital signs as if he or she were receiving type AB-negative packed red blood cells. The blood bank would not need to be called. Blood would not need to be stopped because the blood is compatible with the client's blood type. Benadryl would be given only if the client had an allergic reaction.
23. The nurse is teaching a client who was recently diagnosed with thrombocytopenia. Which instruction does the nurse include in this client's discharge teaching? a." Drink at least 3 liters of fluid each day." b. " Use a soft-bristled toothbrush." c." Avoid blowing your nose." d." Use only aspirin when having pain."
ANS: B Decreased platelet counts increase the risk for prolonged bleeding, even with slight injury. Fluid intake will not affect the platelet count. The client can blow his or her nose if necessary but should be instructed to do so gently. Aspirin should be avoided because it can cause an even greater risk of bleeding.
7. The nurse is providing health promotion education to a client who has a family history of leukemia. Which factor does the nurse teach this client to avoid? a.Alcohol consumptionb. Exposure to ionizing radiationc.High-cholesterol dietd.Smoking cigarettes
ANS: B Many genetic and environmental factors are involved in the development of leukemia. Exposure to radiation increases the risk for development of leukemia, particularly acute myelogenous leukemia (AML). Although alcohol consumption, high-cholesterol diet, and smoking are not healthy behaviors, they do not increase the risk for leukemia.
14. The nurse is teaching a client who is being discharged to home after bone marrow transplantation. The client asks, " Why is it so important to protect myself from injury?" How does the nurse respond? a." Injuries put you at high risk for infection." b. " Platelet recovery is slow, which makes you at risk for bleeding." c." Severe trauma could result in rejection of the transplant." d." The medications you are taking will make you bruise easily."
ANS: B Platelets recover more slowly than other blood cells after bone marrow transplantation. Thus the client is still thrombocytopenic at home and remains at risk for excessive bleeding after any trauma or injury. Injured tissue makes a client at risk for infection, and trauma could result in injury to the transplant (but not rejection). However, these are not the best responses to give the client. A steroid regimen may make a client more at risk for bruising, but the most accurate response pertains to platelet recovery
23. The nurse is assessing a client who is 6 hours postoperative from coronary artery bypass graft surgery. The client's mediastinal tubes are not draining. Which action does the nurse implement at this time? a.Replace the drainage tubing.b. Check for kinks in the tubing. c.Irrigate the tubing with normal saline.d.Document the finding
ANS: B Sudden cessation of mediastinal drainage could result in cardiac tamponade from accumulation of blood around the heart. If the tubing is kinked, this can be addressed quickly. If the tubing is not kinked, immediate notification of the provider is required. The other actions do not correctly address the problem.
19. The nurse is caring for a client who is receiving heparin therapy. How does the nurse evaluate the therapeutic effect of the therapy? a.Evaluate platelets.b. Monitor the partial thromboplastin time (PTT).c.Assess bleeding time.d.Monitor fibrin degradation products
ANS: B The PTT assesses the intrinsic clotting cascade. Heparin therapy is monitored by the PTT. Platelets are monitored by the platelet count laboratory value, bleeding time evaluates vascular and platelet activity during hemostasis, and fibrin degradation products help assess for fibrinolysis
22. The nurse is assessing a client who has a serum potassium level of 4.5 mEq/L after coronary artery bypass graft (CABG) surgery. Which action does the nurse take? a.Notify the health care provider.b. Document the finding.c.Administer prescribed diuretics.d.Administer prescribed potassium replacements.
ANS: B The client who is postoperative from a CABG is at risk for hypokalemia from hemodilution, nasogastric suction, or diuretic therapy. Therefore, the potassium level is maintained between 4 and 5 mEq/L to avoid dysrhythmias. This value is at the desired level for this client. The finding requires documentation only.
9. The nurse is planning care for a client who has a platelet count of 30,000/mm3. Which intervention does the nurse include in this client's plan of care? a.Oxygen by nasal cannulab. Bleeding Precautionsc.Isolation Precautions d.Vital signs every 4 hours
ANS: B The normal platelet count ranges between 150,000 and 400,000/mm3. This client is at extreme risk for bleeding. Although it is necessary to notify the provider, the nurse would first protect the client by instituting Bleeding Precautions. The other interventions are not related to the low platelet count.
1. The nurse helps to ambulate a client who has anemia. Which clinical manifestation indicates that the client is not tolerating the activity? a.Blood pressure of 120/90 mm Hgb. Heart rate of 110 beats/minc.Pulse oximetry reading of 95%d.Respiratory rate of 20 breaths/min
ANS: B The red blood cells contain thousands of hemoglobin molecules. The most important feature of hemoglobin is its ability to combine loosely with oxygen. A low hemoglobin level can cause decreased oxygenation to the tissues, thus causing a compensatory increase in heart rate. The other options are close to normal range and are not indicative of not tolerating this activity.
9. The nurse is planning care for a client who has leukemia. Which intervention does the nurse include in the plan of care to prevent fatigue? a.Arrange for a family member to stay with the client.b. Plan care for times when the client has the most energy.c.Schedule for daily physicals and occupational therapy.d.Plan all activities to occur in the morning to allow for afternoon naps
ANS: B With leukemia, energy management is needed to help conserve the client's energy. Care should be scheduled when the client has the most energy. This client may not have the most energy in the morning. If the benefit of an activity such as physical or occupational therapy is less than its worsening of fatigue, it may be postponed. The nurse should limit the number of visitors and interruptions by visitors, as appropriate.
3. The nurse administers intravenous dobutamine (Dobutrex) to a client who has heart failure. Which clinical manifestations indicate that the client's status is improving? (Select all that apply.) a.Decreased heart rateb. Increased heart ratec.Increased contractilityd.Decreased contractilitye.Increased respiratory rate
ANS: B, C Dobutamine is a positive inotropic agent that works by stimulating beta-adrenergic receptor sites. The result of this stimulation is an increase in the rate and force of the myocardial contraction. Dobutamine has no effect on respiratory rate.
1. A nursing student learns about modifiable risk factors for coronary artery disease. Which factors does this include? (Select all that apply.) a.Age b.Hypertension c.Obesity d. Smoking e.Stress
ANS: B, C, D, E Hypertension, obesity, smoking, and excessive stress are all modifiable risk factors for coronary artery disease. Age is a nonmodifiable risk factor.
3. A nursing student learns that many drugs can impair the immune system. Which drugs does this include? (Select all that apply.) a.Acetaminophen (Tylenol) b.Amphotericin B (Fungizone) c.Ibuprofen (Motrin) d.Metformin (Glucophage) e.Nitrofurantoin (Macrobid)
ANS: B, C, E Amphotericin B, ibuprofen, and nitrofurantoin all can disrupt the hematologic (immune) system. Acetaminophen and metformin do not.
4. A nurse works in a gerontology clinic. What age-related changes cause the nurse to alter standard assessment techniques from those used for younger adults? (Select all that apply.) a.Dentition deteriorates with more cavities. b.Nail beds may be thickened or discolored. c.Progressive loss of hair occurs with age. d.Sclerae begin to turn yellow or pale. e.Skin becomes dry as the client ages.
ANS: B, C, E Common findings in older adults include thickened or discolored nail beds, dry skin, and thinning hair. The nurse adapts to these changes by altering assessment techniques. Having more dental caries and changes in the sclerae are not normal age-related changes.
3. A client has Hodgkin's lymphoma, Ann Arbor stage Ib. For what manifestations should the nurse assess the client? (Select all that apply.) a.Headaches b.Night sweats c.Persistent fever d.Urinary frequency e.Weight loss
ANS: B, C, E In this stage, the disease is located in a single lymph node region or a single non-lymph node site. The client displays night sweats, persistent fever, and weight loss. Headache and urinary problems are not related.
1. The nurse is preparing to administer transfusion therapy to a client. Which interventions does the nurse implement before starting the infusion? (Select all that apply.) a.Confirm the client's room number with the blood tag.b. Check the client's ABO and Rh types with the blood tag.c.Place a 20-gauge needle or larger in the client's forearm.d.Obtain the client's pulse oximetry reading.e.Assess the client's temperature
ANS: B, C, E Before giving any transfusion therapy, two nurses must examine the blood tag and the requisition slip to ensure that the ABO and Rh types are compatible. The client's room number is not an acceptable form of identification. A larger needle (at least a 20-gauge needle) should be used, and blood pressure, pulse, respirations, and temperature should be obtained. Obtaining an oxygen saturation reading is unnecessary.
7. A client with chronic anemia has had many blood transfusions. What medications does the nurse anticipate teaching the client about adding to the regimen? (Select all that apply.) a.Azacitidine (Vidaza) b.Darbepoetin alfa (Aranesp) c.Decitabine (Dacogen) d.Epoetin alfa (Epogen) e.Methylprednisolone (Solu-Medrol)
ANS: B, D Darbepoetin alfa and epoetin alfa are both red blood cell colony-stimulating factors that will help increase the production of red blood cells. Azacitidine and decitabine are used for myelodysplastic syndromes. Methylprednisolone is a steroid and would not be used for this problem.
4. A client is 1 day postoperative after a coronary artery bypass graft. What nonpharmacologic comfort measures does the nurse include when caring for this client? (Select all that apply.) a.Administer pain medication before ambulating. b.Assist the client into a position of comfort in bed. c.Encourage high-protein diet selections. d.Provide complementary therapies such as music. e.Remind the client to splint the incision when coughing.
ANS: B, D, E Nonpharmacologic comfort measures can include positioning, complementary therapies, and splinting the chest incision. Medications are not nonpharmacologic. Food choices are not comfort measures.
2. The nurse is monitoring the electrocardiogram (ECG) of a client who has a myocardial infarction. Which changes does the nurse expect to see in the ECG tracing? (Select all that apply.) a.ST-segment depressionb. T-wave inversionc.Normal Q wavesd.ST-segment elevatione.T-wave elevationf.Abnormal Q wave
ANS: B, D, F When myocardial infarction occurs, the changes usually seen on an ECG tracing are ST-segment elevation, T-wave inversion, and an abnormal Q wave
1. A client is receiving an infusion of tissue plasminogen activator (t-PA). The nurse assesses the client to be disoriented to person, place, and time. What action by the nurse is best? a.Assess the client's pupillary responses. b.Request a neurologic consultation. c.Stop the infusion and call the provider. d. Take and document a full set of vital signs.
ANS: C A change in neurologic status in a client receiving t-PA could indicate intracranial hemorrhage. The nurse should stop the infusion and notify the provider immediately. A full assessment, including pupillary responses and vital signs, occurs next. The nurse may or may not need to call a neurologist.
7. A nurse is caring for four clients. After reviewing today's laboratory results, which client should the nurse see first? a.Client with an international normalized ratio of 2.8 b.Client with a platelet count of 128,000/mm3 c.Client with a prothrombin time (PT) of 28 seconds d.Client with a red blood cell count of 5.1 million/μL
ANS: C A normal PT is 11 to 12.5 seconds. This client is at high risk of bleeding. The other values are within normal limits.
24. The provider requests the nurse start an infusion of an inotropic agent on a client. How does the nurse explain the action of these drugs to the client and spouse? a."It constricts vessels, improving blood flow." b."It dilates vessels, which lessens the work of the heart." c."It increases the force of the heart's contractions." d."It slows the heart rate down for better filling."
ANS: C A positive inotrope is a medication that increases the strength of the heart's contractions. The other options are not correc
10. A client in the cardiac stepdown unit reports severe, crushing chest pain accompanied by nausea and vomiting. What action by the nurse takes priority? a.Administer an aspirin. b.Call for an electrocardiogram (ECG). c.Maintain airway patency. d.Notify the provider.
ANS: C Airway always is the priority. The other actions are important in this situation as well, but the nurse should stay with the client and ensure the airway remains patent (especially if vomiting occurs) while another person calls the provider (or Rapid Response Team) and facilitates getting an ECG done. Aspirin will probably be administered, depending on the provider's prescription and the client's current medications
13. A nurse is caring for a young male client with lymphoma who is to begin treatment. What teaching topic is a priority? a.Genetic testing b.Infection prevention c.Sperm banking d. Treatment options
ANS: C All teaching topics are important to the client with lymphoma, but for a young male, sperm banking is of particular concern if the client is going to have radiation to the lower abdomen or pelvis.
22. A client has a sickle cell crisis with extreme lower extremity pain. What comfort measure does the nurse delegate to the unlicensed assistive personnel (UAP)? a.Apply ice packs to the client's legs. b.Elevate the client's legs on pillows. c.Keep the lower extremities warm. d.Place elastic bandage wraps on the client's legs.
ANS: C During a sickle cell crisis, the tissue distal to the occlusion has decreased blood flow and ischemia, leading to pain. Due to decreased blood flow, the client's legs will be cool or cold. The UAP can attempt to keep the client's legs warm. Ice and elevation will further decrease perfusion. Elastic bandage wraps are not indicated and may constrict perfusion in the legs.
13. A client has an intra-arterial blood pressure monitoring line. The nurse notes bright red blood on the client's sheets. What action should the nurse perform first? a.Assess the insertion site. b.Change the client's sheets. c.Put on a pair of gloves. d.Assess blood pressure.
ANS: C For the nurse's safety, he or she should put on a pair of gloves to prevent blood exposure. The other actions are appropriate as well, but first the nurse must don a pair of gloves.
3. The nurse observes yellow-tinged sclera in a client with dark skin. Based on this assessment finding, what does the nurse do next? a.Assess the client's pulses.b. Examine the soles of the client's feet.c.Inspect the client's hard palate.d.Auscultate the client's lung sounds.
ANS: C Jaundice can best be observed in clients with dark skin by inspecting the oral mucosa, especially the hard palate, for yellow discoloration. Because sclera may have subconjunctival fat deposits that show a yellow hue, and because foot calluses may appear yellow, neither of these areas should be used to assess for jaundice. The client's pulse and lung sounds have no correlation with an assessment of jaundice
3. The community health nurse assesses clients at a health fair. Which statement assists the nurse to identify modifiable risk factors in clients with coronary artery disease? a." Would you please state your full name and birth date?" b. " Have you ever had an exercise tolerance stress test?" c." In what activities do you participate on a daily basis?" d." Does anyone in your family have a history of heart disease?"
ANS: C Modifiable risk factors can be altered or controlled. Cigarette smoking and a sedentary lifestyle are examples of behaviors that are modifiable. Nonmodifiable factors are personal elements that cannot be altered or controlled (e.g., age, gender, family history). A stress test would not provide any information about risk factors
22. A client had an acute myocardial infarction. What assessment finding indicates to the nurse that a significant complication has occurred? a.Blood pressure that is 20 mm Hg below baseline b.Oxygen saturation of 94% on room air c.Poor peripheral pulses and cool skin d. Urine output of 1.2 mL/kg/hr for 4 hours
ANS: C Poor peripheral pulses and cool skin may be signs of impending cardiogenic shock and should be reported immediately. A blood pressure drop of 20 mm Hg is not worrisome. An oxygen saturation of 94% is just slightly below normal. A urine output of 1.2 mL/kg/hr for 4 hours is normal.
10. A nurse is caring for a client who is about to receive a bone marrow transplant. To best help the client cope with the long recovery period, what action by the nurse is best? a.Arrange a visitation schedule among friends and family. b.Explain that this process is difficult but must be endured. c.Help the client find things to hope for each day of recovery. d.Provide plenty of diversionary activities for this time.
ANS: C Providing hope is an essential nursing function during treatment for any disease process, but especially during the recovery period after bone marrow transplantation, which can take up to 3 weeks. The nurse can help the client look ahead to the recovery period and identify things to hope for during this time. Visitors are important to clients, but may pose an infection risk. Telling the client the recovery period must be endured does not acknowledge his or her feelings. Diversionary activities are important, but not as important as instilling hope.
23. A client presents to the emergency department with an acute myocardial infarction (MI) at 1500 (3:00 PM). The facility has 24-hour catheterization laboratory abilities. To meet The Joint Commission's Core Measures set, by what time should the client have a percutaneous coronary intervention performed? a.1530 (3:30 PM) b.1600 (4:00 PM) c.1630 (4:30 PM) d. 1700 (5:00 PM)
ANS: C The Joint Commission's Core Measures set for MI includes percutaneous coronaryintervention within 90 minutes of diagnosis of myocardial infarction. Therefore, the client should have a percutaneous coronary intervention performed no later than 1630 (4:30 PM)
21. A client has thrombocytopenia. What client statement indicates the client understands self-management of this condition? a."I brush and use dental floss every day." b."I chew hard candy for my dry mouth." c."I usually put ice on bumps or bruises." d."Nonslip socks are best when I walk."
ANS: C The client should be taught to apply ice to areas of minor trauma. Flossing is not recommended. Hard foods should be avoided. The client should wear well-fitting shoes when ambulating.
27. A client has been treated for a deep vein thrombus and today presents to the clinic with petechiae. Laboratory results show a platelet count of 42,000/mm3. The nurse reviews the client's medication list to determine if the client is taking which drug? a.Enoxaparin (Lovenox) b.Salicylates (aspirin) c.Unfractionated heparin d.Warfarin (Coumadin)
ANS: C This client has manifestations of heparin-induced thrombocytopenia. Enoxaparin, salicylates, and warfarin do not cause this condition.
27. A client who is receiving a unit of red blood cells begins to report chest and lower back pain. Which action does the nurse take first? a.Administer morphine sulfate 1 mg IV.b. Assess the level of the pain.c.Stop the transfusion.d.Reposition the client on the right side.
ANS: C A hemolytic transfusion reaction is caused by blood type or Rh incompatibility. When blood containing antigens different from the client's own antigens is infused, antigen-antibody complexes are formed in the client's blood. Manifestations include low back pain and chest pain, and the transfusion should be discontinued immediately. The other actions are not the priority
1. The registered nurse is assigning a practical nurse to care for a client who has leukemia. Which instruction does the registered nurse provide to the practical nurse when delegating this client's care? a.Evaluate the amount of protein the client eats.b. Assess the client's roommate for symptoms of infection.c.Perform effective hand hygiene frequently.d.Wear a mask when entering the room.
ANS: C A major objective in caring for the client with leukemia is protection from infection. Frequent handwashing is of the utmost importance. If at all possible, the client should be in a private room. Masks are worn by anyone who has an upper respiratory tract infection. The client may be on a " minimal bacteria diet." Protein is not a factor in this diet.
12. The nurse assesses that a client has a smooth, beefy red tongue. Which intervention does the nurse implement for this client? a.Administer prescribed oral iron supplements.b. Monitor the daily white blood cell count.c.Provide a diet high in green leafy vegetables.d.Perform more frequent mouth care.
ANS: C A smooth, beefy red tongue could signify glossitis, which is seen with vitamin B12 deficiency. Green leafy vegetables are high in vitamin B12. Iron supplements would be used with iron deficiency anemia. The red blood cell count is what is affected by vitamin B12 deficiency—not the white blood cell count. The beefy red tongue is caused by the vitamin deficiency, not by poor mouth care.
18. A client who is post percutaneous transluminal coronary angioplasty (PTCA) reports severe chest pain. Which action does the nurse take first? a.Administer the prescribed IV morphine.b. Administer the prescribed sublingual nitroglycerin.c.Assess the client's vital signs and notify the health care provider.d.Perform an immediate 12-lead ECG.
ANS: C After PTCA, a small percentage of clients experience acute restenosis (closure) of the affected coronary artery. Chest pain similar to that experienced before the procedure may indicate acute restenosis. The client will need to return to the catheterization laboratory to have the procedure repeated and may need stent placement to maintain a patent vessel lumen. The nurse may relieve pain with morphine or nitroglycerin after contacting the provider. The provider may request an ECG
22. A client who has sickle cell anemia is admitted to the hospital. The client reports severe pain. Which action does the nurse take first? a.Administer one unit of packed red blood cells.b. Administer prescribed hydroxyurea (Droxia).c.Begin intravenous fluids at 250 mL/hr.d.Prepare for bone marrow transplantation.
ANS: C All of these are treatments for sickle cell anemia. However, the client in severe pain is likely to be in sickle cell crisis. To prevent further sickling of the red blood cells, adequate hydration of at least 200 mL/hr is needed during a crisis. The other interventions should be implemented after the fluids are started.
24. The nurse is caring for an 80-year-old client who has had coronary artery bypass graft surgery. Which assessment does the nurse prioritize for this client? a.Skinb. Otoscopicc.Mental statusd.Gastrointestinal
ANS: C Assessment of mental status is important because older adults are more likely to experience transient neurologic deficits as compared with younger adults. The other assessments are not a priority for this client.
22. The nurse is preparing a client for a bone biopsy and aspiration. The client asks, "Will this be painful?" How does the nurse respond? a."The procedure is always done under general anesthesia."b. "The biopsy lasts for only 2 minutes."c."There is a chance that you may have pain."d."You can relieve pain with guided imagery."
ANS: C Clients may have pain during this procedure. The type and amount of anesthesia or sedation depend on the physician's preference, the client's preference, and previous experience with bone marrow aspiration. The procedure takes from 5 to 15 minutes. Guided imagery can relieve pain but works well only with some clients.
21. A client is newly diagnosed with sickle cell anemia. Which information does the nurse include in the client's discharge instructions? a." Eat a diet high in iron." b. " Take hydroxyurea (Droxia) every morning." c." Be aware of the early symptoms of crisis." d." Do not use any oral contraceptives."
ANS: C Clients need to know the early symptoms of crisis so that treatment can be started early to prevent pain, complications, and permanent tissue damage. The iron level is not low in sickle cell anemia. Hydroxyurea is used in the hospital during a sickle cell crisis. The use of oral contraceptives is controversial because they may enhance clot formation, predisposing the client to crisis.
20. The nurse is transfusing red blood cells to a client who has sickle cell disease. Which laboratory result indicates that the nurse should discontinue the transfusion? a.Hematocrit level (Hct), 32%b. Hemoglobin S, 88%c.Serum iron level, 300 mcg/dLd.Total white blood cell count, 12,000/mm3
ANS: C Clients with sickle cell disease are anemic but are not iron deficient. Transfusions are prescribed cautiously to prevent iron overload with repeated transfusions. Iron overload damages the heart, liver, and endocrine organs. Monitor the client's serum ferritin, serum iron (Fe), and total iron-binding capacity (TIBC) during transfusion therapy. The other laboratory values should not result in discontinuation of the transfusion by the nurse.
14. The nurse is assessing a client's susceptibility to rejecting a transplanted kidney. Which result does the nurse recognize as increasing the client's chances of rejection? a.Decreased T-lymphocyte helperb. Decreased white blood cell countc.Increased cytotoxic-cytolytic T celld.Increased neutrophil count
ANS: C Cytotoxic-cytolytic T cells function to attack and destroy non-self-cells, specifically virally infected cells and cells from transplanted grafts and organs. A high level of these cells would increase the chances of rejection. Decreased white blood cells would indicate immune suppression. Neutrophils are increased during an infection.
28. The nurse is preparing to transfuse a third unit of red blood cells to a client. Which laboratory result is the nurse most concerned about? a.Fibrinogen level less than 100 mg/dLb. Hematocrit of 30%c.Potassium level of 5.5 mg/dLd.Serum ferritin level of 250 ng/mL
ANS: C Electrolyte imbalance is possible as a result of transfusions, especially with red blood cells or whole blood. Potassium is the main electrolyte inside cells. During transfusion, some cells are damaged and release potassium. Low fibrinogen levels would require transfusion of cryoprecipitate. The client would be a candidate for red blood cell transfusion if his hematocrit level were low, so this would not be a concern for preparation of the red blood cells. The serum ferritin level is normal and is not a matter of concern
5. The emergency department nurse is assessing an 82-year-old client for a potential myocardial infarction. Which clinical manifestation does the nurse monitor for? a.Pain on inspirationb. Posterior wall chest painc.Disorientation or confusiond.Numbness and tingling of the arm
ANS: C In older adults, disorientation or confusion may be the major manifestation of myocardial infarction caused by poor cardiac output. Pain manifestations could also be related to the myocardial infarction. However, the nurse is more concerned about the new onset of disorientation or confusion caused by decreased perfusion.
13. A female client is admitted with the medical diagnosis of anemia. The nurse assesses for which potential cause? a.Diet high in meat and fatb. Daily intake of aspirinc.Heavy mensesd.Smoking history
ANS: C Iron levels can be low because intake of iron is too low, or because loss of iron through bleeding is excessive. A premenopausal woman may be having unusually heavy menses sufficient to cause excessive loss of blood and iron. Smoking and aspirin do not cause iron deficiency. A diet high in meat provides iron.
18. The nurse is caring for a client during a sickle cell crisis. Which intervention does the nurse implement for the client? a.Administer acetaminophen (Tylenol) as needed.b. Administer intravenous fluids to keep the vein open.c.Keep the room temperature at 80° F.d.Transfuse red blood cells (RBCs).
ANS: C Keeping the room warm can be used as a complementary therapy to relieve the pain of a sickle cell crisis. Cold can act as a factor in causing a crisis. Analgesia is an important part of relieving pain. The analgesia routine should be followed on an around-the-clock basis and should consist of IV opioids for severe pain, followed by treatment with oral doses of opioids or NSAIDs. High-volume intravenous fluids should be administered to minimize pain during a sickle cell crisis.
4. The nurse teaches a client who is newly diagnosed with coronary artery disease. Which instruction does the nurse include to minimize complications of this disease? a." Rest is the best medicine at this time. Do not start an exercise program." b. " You are a man; therefore there is nothing you can do to minimize your risks." c." You should talk to your provider about medications to help you quit smoking." d." Decreasing the carbohydrates in your diet will help you lose weight."
ANS: C Modifiable risk factors can be altered or controlled. Cigarette smoking and a sedentary lifestyle are examples of behaviors that are modifiable. Nonmodifiable factors are personal elements that cannot be altered or controlled (e.g., age, gender, family history). The nurse needs to encourage the client to stop smoking because this is a proven risk factor for coronary artery disease development. The nurse should also encourage weight loss and moderate exercise.
6. Eight hours after presentation to the emergency department with reports of substernal chest pain, a client's laboratory results demonstrate myoglobin levels of 55 ng/mL. What does the nurse do next? a.Prepare the client for an emergency coronary bypass graft surgery.b. Administer nitroglycerin to prevent further myocardial cell death. c.Assess the client to identify another potential cause of the chest pain.d.Provide client education related to complications of myocardial infarctions.
ANS: C Myoglobin is a heme protein found in skeletal and cardiac muscle. With myocardial injury, myoglobin levels rise within 3 to 6 hours. If myoglobin levels have not risen within that time, the client has not experienced a myocardial infarction. The nurse should assess the client to identify a potential cause for the chest pain, besides an MI.
10. The nurse is teaching a client with vitamin B12 deficiency anemia to eat a diet high in this vitamin. Which meal selected by the client indicates that the client correctly understands the prescribed diet? a.Baked chicken breast, mashed potatoes, glass of milkb. Eggplant parmesan, cottage cheese, iced teac.Fried liver and onions, orange juice, spinach saladd.Fettuccine alfredo, green salad, glass of red wine
ANS: C Organ meats and leafy green vegetables have the highest content of vitamin B12. The other selections do not indicate understanding of the teaching on diet.
5. The nurse is teaching a client who is being discharged after stem cell transplantation. Which instruction does the nurse include in this client's discharge teaching? a.Eat a diet high in fruits and vegetables.b. Ask your provider to administer a rubella vaccination.c.Wash your hands frequently.d.Participate in physical therapy every day.
ANS: C Protecting the client from infection at home is just as important as it was during hospitalization for a client who has had stem cell transplantation. Hand hygiene is the best protection against infection. Salads, raw fruits, and live vaccinations (such as rubella) are contraindicated in a client who has a risk for infection. Energy management is important; therefore activities such as physical therapy may need to be postponed
11. A client who presented with an acute myocardial infarction is prescribed thrombolytic therapy. The client had a stroke 1 month ago. Which action does the nurse take? a.Administer the medication as prescribed.b. Perform a CT scan before administering the medication.c.Contact the health care provider to discontinue the prescribed therapy.d.Administer the therapy with a normal saline bolus.
ANS: C Recent stroke (within 2 months) is an absolute contraindication to thrombolytic therapy. The nurse should not give the medication under any conditions. The provider must be notified and made aware of the client's stroke history. None of the other options are appropriate.
25. The nurse is planning discharge education for a client after coronary artery bypass graft surgery. Which instruction does the nurse include in this client's teaching? a." Remember to drink at least 3 liters of fluid daily." b. " You should abstain from sexual activity for 6 months." c." Take your pulse before, midway through, and after exercising." d." Stop taking your antihyperlipidemic medication at this time."
ANS: C The client is instructed to begin a walking program that gradually lengthens in distance. The client is advised to take his or her pulse before exercising, midway through exercising, and after exercising. The client should stop exercising if the target rate is exceeded or if angina develops. The client should not take in large quantities of fluids or stop taking antihyperlipidemic medications. The client does not need to abstain from sexual activity.
15. The nurse is caring for a client who is receiving chemotherapy for cancer. Which intervention does the nurse implement for this client? a.Assess the client's fibrinogen level.b. Administer the prescribed iron.c.Maintain strict Standard Precautions.d.Monitor the client's pulse oximetry.
ANS: C The client who is receiving chemotherapy drugs that suppress the bone marrow will be at risk for a decreased white blood cell (WBC) count and infection. The nurse will be most therapeutic by adhering to Standard Precautions to prevent infection, such as handwashing. The nurse will not expect the fibrinogen level to be affected by this therapy. Iron is not typically administered with chemotherapy because this is bone marrow suppression, so the administration of epoetin (Epogen) or filgrastim (Neupogen) is most effective. Monitoring the pulse oximetry is part of routine care and probably would not need to be done continuously.
23. The nurse is caring for four clients with hematologic-type problems. Which client does the nurse prioritize to see first? a.18-year-old female with decreased protein levelsb. 36-year-old male with increased lymphocytesc.60-year-old female with decreased erythropoietin d.82-year-old male with an increased thromboxane level
ANS: C The kidney releases more erythropoietin when tissue oxygenation levels are low. This growth factor then stimulates the bone marrow to increase red blood cell (RBC) production, which improves tissue oxygenation and prevents hypoxia. Hypoxia causes the body to increase its respiratory rate to overcome decreased oxygenation of the tissues. All these clients are important, but the woman with decreased erythropoietin takes priority because of her risk for hypoxia.
16. The nurse is performing an admission assessment on a 46-year-old client, who states, "I have been drinking a 12-pack of beer every day for the past 20 years." Which laboratory abnormality does the nurse correlate with this history? a.Decreased white blood cell (WBC) countb. Decreased bleeding timec.Elevated prothrombin time (PT)d.Elevated red blood cell (RBC) count
ANS: C The liver is the site for production of prothrombin and most of the blood-clotting factors. If the liver is damaged because of chronic alcoholism, it is unable to produce these clotting factors. Therefore, the PT could become elevated, which would reflect deficiency of some clotting factors. The WBC would not be elevated in this situation because no infection is present. Bleeding time would likely increase. The client's RBC count most likely would not be affected unless the client was bleeding, in which case it would decrease.
15. The nurse is assessing a client who has left ventricular failure secondary to a myocardial infarction. Which clinical manifestation of poor organ perfusion does the nurse monitor for in this client? a.Headacheb. Hypertensionc.Urine output of less than 30 mL/hrd.Heart rate of 55 to 60 beats/min
ANS: C The nurse should remain alert for signs of poor organ perfusion that are the result of decreased cardiac output. When the kidneys are not well perfused, urine output drops to less than 30 mL/hr. Other signs include changes in mental status; cool, clammy extremities with decreased or absent pulses; fatigue; and recurrent chest pain. The other manifestations do not indicate poor organ perfusion.
6. The nurse prepares to administer a blood transfusion to a client. Which means of identification does the nurse use to ensure that the blood is administered to the correct client? a.Ask the client whether his or her name is the one on the blood product tag.b. Ask the client's spouse if the client is supposed to have a transfusion. c.Compare the name and ID number on the blood product tag with the name and ID number on the client's ID band.d.Compare the unit and room number of the client with the unit and room number listed on the blood product tag
ANS: C The safest way to determine whether the blood product is to be given to the correct client is to check the client's hospital ID band and compare the information on it with that on the blood product tag. The room and unit numbers are never considered as means of positive identification. Asking the client who he or she is might result in an error if the client is confused. Similarly, a visitor cannot be assumed to know whether this is the client to have the blood transfusion.
11. The nurse is teaching a client who has iron deficiency anemia. Which food choice indicates that the client correctly understands the teaching? a.Chickenb. Orangesc.Steakd.Tomatoes
ANS: C Treatment for iron deficiency anemia involves increasing oral intake of iron from food sources. Foods high in iron include red meat, organ meat, kidney beans, leafy green vegetables, and raisins
5. A client who has a chronic vitamin B12 deficiency is admitted to the hospital. When obtaining the client's health history, which priority question does the nurse ask this client? a."Are you having any pain?"b. "Are you having blood in your stools?"c."Do you notice any changes in your memory?"d."Do you bruise easily?"
ANS: C Vitamin B12 deficiency impairs cerebral, olfactory, spinal cord, and peripheral nerve function. Severe chronic deficiency may cause permanent neurologic degeneration. The other options are not symptoms of vitamin B12 deficiency.
2. The nurse is teaching a client who is scheduled to undergo allogeneic bone marrow transplantation. Which statements indicate that the client correctly understands the teaching? (Select all that apply.) a." The surgeon will insert the marrow into my femur bone." b. " Until the marrow transplant takes, I can have visitors." c." The transplant does not start working immediately." d." I will need chemotherapy before my transplant." e." Radiation treatments will begin 2 days after transplantation."
ANS: C, D Engraftment, or the successful take of transplanted cells, takes anywhere from 8 to 28 days, depending on the type of cell transplantation. For donated marrow or stem cells to work, the client will require large doses of chemotherapy before transplantation. The client will not require radiation after the transplant. Transplanted marrow is delivered intravenously. It is not placed into any bone. The client is at risk for infection until the bone marrow begins to produce white blood cells. Therefore visitors should be limited to prevent infection to the client.
10. A client has received a bone marrow transplant and is waiting for engraftment. What actions by the nurse are most appropriate? (Select all that apply.) a.Not allowing any visitors until engraftment b.Limiting the protein in the client's diet c.Placing the client in protective precautions d.Teaching visitors appropriate hand hygiene e.Telling visitors not to bring live flowers or plants
ANS: C, D, E The client waiting for engraftment after bone marrow transplant has no white cells to protect him or her against infection. The client is on protective precautions and visitors are taught hand hygiene. No fresh flowers or plants are allowed due to the standing water in the vase or container that may harbor organisms. Limiting protein is not a healthy option and will not promote engraftment.
4. A client has a platelet count of 25,000/mm3. What actions does the nurse delegate to the unlicensed assistive personnel (UAP)? (Select all that apply.) a.Assist with oral hygiene using a firm toothbrush. b.Give the client an enema if he or she is constipated. c.Help the client choose soft foods from the menu. d.Shave the male client with an electric razor. e.Use a lift sheet when needed to re-position the client.
ANS: C, D, E This client has thrombocytopenia and requires bleeding precautions. These include oral hygiene with a soft-bristled toothbrush or swabs, avoiding rectal trauma, eating soft foods, shaving with an electric razor, and using a lift sheet to re-position the client.
1. A nursing student wants to know why clients with chronic obstructive pulmonary disease tend to be polycythemic. What response by the nurse instructor is best? a. It is due to side effects of medications for bronchodilation. b. It is from overactive bone marrow in response to chronic disease. c. It combats the anemia caused by an increased metabolic rate. d. It compensates for tissue hypoxia caused by lung disease.
ANS: D In response to hypoxia, more red blood cells are made so more oxygen can be carried and delivered to tissues. This is a physiologic process in response to the disease; it is not a medication side effect, the result of overactive bone marrow, or a response to anemia.
15. A client with multiple myeloma demonstrates worsening bone density on diagnostic scans. About what drug does the nurse plan to teach this client? a.Bortezomib (Velcade) b.Dexamethasone (Decadron) c.Thalidomide (Thalomid) d.Zoledronic acid (Zometa)
ANS: D All the options are drugs used to treat multiple myeloma, but the drug used specifically for bone manifestations is zoledronic acid (Zometa), which is a bisphosphonate. This drug class inhibits bone resorption and is used to treat osteoporosis as well.
24. A client with sickle cell disease (SCD) takes hydroxyurea (Droxia). The client presents to the clinic reporting an increase in fatigue. What laboratory result should the nurse report immediately? a.Hematocrit: 25% b.Hemoglobin: 9.2 mg/dL c.Potassium: 3.2 mEq/L d.White blood cell count: 38,000/mm
ANS: D Although individuals with SCD often have elevated white blood cell (WBC) counts, this extreme elevation could indicate leukemia, a complication of taking hydroxyurea. The nurse should report this finding immediately. Alternatively, it could indicate infection, a serious problem for clients with SCD. Hematocrit and hemoglobin levels are normally low in people with SCD. The potassium level, while slightly low, is not as worrisome as the WBCs.
26. The nurse is assessing a client with a history of heart failure who is receiving a unit of packed red blood cells. The client's respiratory rate is 33 breaths/min and blood pressure is 140/90 mm Hg. Which action does the nurse take first? a.Administer prescribed diphenhydramine (Benadryl).b. Continue to monitor the client's vital signs.c.Stop the infusion of packed red blood cells.d.Slow the infusion rate of the transfusion.
ANS: D Circulatory overload can occur when a blood product is infused too quickly. Adults with a history of heart failure are at risk for this. Management of this complication can be achieved by infusing the blood products more slowly. The client is not having an allergic reaction to the blood; therefore the blood should not be stopped nor should diphenhydramine be administered
11. A nursing student is struggling to understand the process of graft-versus-host disease. What explanation by the nurse instructor is best? a."Because of immunosuppression, the donor cells take over." b."It's like a transfusion reaction because no perfect matches exist." c."The client's cells are fighting donor cells for dominance." d."The donor's cells are actually attacking the client's cells."
ANS: D Graft versus host disease is an autoimmune-type process in which the donor cells recognize the client's cells as foreign and begin attacking them. The other answers are not accurate.
8. A nursing student is caring for a client with leukemia. The student asks why the client is still at risk for infection when the client's white blood cell count (WBC) is high. What response by the registered nurse is best? a."If the WBCs are high, there already is an infection present." b."The client is in a blast crisis and has too many WBCs." c."There must be a mistake; the WBCs should be very low." d. "Those WBCs are abnormal and don't provide protection."
ANS: D In leukemia, the WBCs are abnormal and do not provide protection to the client against infection. The other statements are not accurate
20. The nurse is assessing a client who had percutaneous transluminal coronary angioplasty (PTCA) 1 hour ago. Which complication does the nurse monitor for? a.Hypertensive crisisb. Hyperkalemiac.Infectiond.Bleeding
ANS: D In the first few postprocedure hours, the nurse monitors for complications such as bleeding from the insertion site, hypotension, acute closure of the vessel, dye reaction, hypokalemia, and dysrhythmias. The other problems are not complications in the immediate post-PTCA period.
5. A client undergoing hemodynamic monitoring after a myocardial infarction has a right atrial pressure of 0.5 mm Hg. What action by the nurse is most appropriate? a.Level the transducer at the phlebostatic axis. b.Lay the client in the supine position. c.Prepare to administer diuretics. d.Prepare to administer a fluid bolus.
ANS: D Normal right atrial pressures are from 1 to 8 mm Hg. Lower pressures usually indicate hypovolemia, so the nurse should prepare to administer a fluid bolus. The transducer should remain leveled at the phlebostatic axis. Positioning may or may not influence readings. Diuretics would be contraindicated
8. A client is in the preoperative holding area prior to an emergency coronary artery bypass graft (CABG). The client is yelling at family members and tells the doctor to "just get this over with" when asked to sign the consent form. What action by the nurse is best? a.Ask the family members to wait in the waiting area. b.Inform the client that this behavior is unacceptable. c.Stay out of the room to decrease the client's stress levels. d.Tell the client that anxiety is common and that you can help.
ANS: D Preoperative fear and anxiety are common prior to cardiac surgery, especially in emergent situations. The client is exhibiting anxiety, and the nurse should reassure the client that fear is common and offer to help. The other actions will not reduce the client's anxiety.
9. A client is having a radioisotopic imaging scan. What action by the nurse is most important? a.Assess the client for shellfish allergies. b.Place the client on radiation precautions. c.Sedate the client before the scan. d.Teach the client about the procedure.
ANS: D The nurse should ensure that teaching is done and the client understands the procedure. Contrast dye is not used, so shellfish/iodine allergies are not related. The client will not be radioactive and does not need radiation precautions. Sedation is not used in this procedure.
25. A nurse is assessing a client who had a myocardial infarction. Upon auscultating heart sounds, the nurse hears the following sound. What action by the nurse is most appropriate? (Click the media button to hear the audio clip.) a.Assess for further chest pain. b.Call the Rapid Response Team. c.Have the client sit upright. d. Listen to the client's lung sounds.
ANS: D The sound the nurse hears is an S3 heart sound, an abnormal sound that may indicate heart failure. The nurse should next assess the client's lung sounds. Assessing for chest pain is not directly related. There is no indication that the Rapid Response Team is needed. Having the client sit up will not change the heart sound
28. The nurse assesses a client's oral cavity and makes the discovery shown in the photo below: What action by the nurse is most appropriate? a.Encourage the client to have genetic testing. b.Instruct the client on high-fiber foods. c.Place the client in protective precautions. d. Teach the client about cobalamin therapy.
ANS: D This condition is known as glossitis, and is characteristic of B12 anemia. If the anemia is a pernicious anemia, it is treated with cobalamin. Genetic testing is not a priority for this condition. The client does not need high-fiber foods or protective precautions.
12. The nurse is preparing to change a client's sternal dressing. What action by the nurse is most important? a.Assess vital signs. b.Don a mask and gown. c.Gather needed supplies. d.Perform hand hygiene.
ANS: D To prevent a sternal wound infection, the nurse washes hands or performs hand hygiene as a priority. Vital signs do not necessarily need to be assessed beforehand. A mask and gown are not needed. The nurse should gather needed supplies, but this is not the priority.
19. A nurse is preparing to hang a blood transfusion. Which action is most important? a.Documenting the transfusion b.Placing the client on NPO status c.Placing the client in isolation d.Putting on a pair of gloves
ANS: D To prevent bloodborne illness, the nurse should don a pair of gloves prior to hanging the blood. Documentation is important but not the priority at this point. NPO status and isolat ion are not needed.
16. A client with autoimmune idiopathic thrombocytopenic purpura (ITP) has had a splenectomy and returned to the surgical unit 2 hours ago. The nurse assesses the client and finds the abdominal dressing saturated with blood. What action is most important? a.Preparing to administer a blood transfusion b.Reinforcing the dressing and documenting findings c.Removing the dressing and assessing the surgical site d.Taking a set of vital signs and notifying the surgeon
ANS: D While some bloody drainage on a new surgical dressing is expected, a saturated dressing is not. This client is already at high risk of bleeding due to the ITP. The nurse should assess vital signs for shock and notify the surgeon immediately. The client may or may not need a transfusion. Reinforcing the dressing is an appropriate action, but the nurse needs to do more than document afterward. Removing the dressing increases the risk of infection; plus, it is not needed since the nurse knows where the bleeding is coming from.
5. A hospitalized client has a platelet count of 58,000/mm3. What action by the nurse is best? a.Encourage high-protein foods. b.Institute neutropenic precautions. c.Limit visitors to healthy adults. d.Place the client on safety precautions.
ANS: D With a platelet count between 40,000 and 80,000/mm3, clients are at risk of prolonged bleeding even after minor trauma. The nurse should place the client on safety precautions. High-protein foods, while healthy, are not the priority. Neutropenic precautions are not needed as the client's white blood cell count is not low. Limiting visitors would also be more likely related to a low white blood cell count.
24. The nurse is assessing the following laboratory results of a client before discharge. Which instruction does the nurse include in this client's discharge teaching plan? Test Result Hemoglobin 15 g/dL Hematocrit 45% White blood cell (WBC) count 2000/mm3Platelet count 250,000/mm3a."Avoid contact sports."b. "Do not take any aspirin."c."Eat a diet high in iron."d."Perform good hand hygiene."
ANS: D A normal WBC count is 5000 to 10,000/mm3. A white blood cell count of 2000/mm3 is low and makes this client at risk for infection. Good handwashing technique is the best way to prevent the transmission of infection. The other laboratory results are all within normal limits.
2. The nurse is teaching a client who has sickle cell disease and was admitted for splenomegaly and abdominal pain. Which instruction does the nurse include in the client's discharge teaching? a." Avoid drinking large amounts of fluids." b. " Eat six small meals daily instead of large meals." c." Engage in aerobic exercise 3 days a week." d." Receive a yearly influenza vaccination."
ANS: D Abdominal pain and a palpable spleen could indicate blood trapping in the spleen. Over time, the spleen may become nonfunctional, which makes the client at risk for infection. An annual influenza vaccination helps prevent infection. A client with sickle cell disease should not become dehydrated or engage in strenuous physical activity because this could precipitate a crisis. Eating smaller meals has no impact on sickle cell disease or infection.
2. The nurse is assessing a client with liver failure. Which assessment is the highest priority for this client? a.Auscultation for bowel soundsb. Assessing for deep vein thrombosisc.Monitoring of blood pressure hourlyd.Assessing for signs of bleeding
ANS: D All these options are important in assessment of the client, but the most important action is assessment for signs of bleeding. The liver is the site of production of prothrombin and most of the blood-clotting factors. Clients with liver failure run a high risk of having problems with bleeding.
24. The nurse is teaching a client who has myelodysplastic syndrome. Which instruction does the nurse include in this client's teaching? a." Rise slowly when getting out of bed." b. " Drink at least 3 liters of liquids per day." c." Wear gloves and socks outdoors in cool weather." d." Use a soft-bristled toothbrush."
ANS: D Myelodysplastic syndrome is a group of disorders that includes anemia, neutropenia, and thrombocytopenia. Because of low platelets, the client is at risk for bleeding. Using a soft-bristled toothbrush minimizes trauma to the gums and prevents bleeding. The other instructions are not appropriate for this syndrome.
13. The nurse is assessing a client who has been prescribed a nonselective beta-blocking agent. Which adverse effect does the nurse monitor for in this client? a.Headacheb. Postural hypotensionc.Nonproductive coughd.Wheezing
ANS: D Nonselective beta blockers can cause bronchoconstriction and impair respiratory effort. Clients with pre-existing pulmonary problems should not take nonselective beta-blocking agents. Clients who develop bronchoconstriction should have their therapy changed. The other manifestations are not adverse effects of this medication.
16. The nurse is caring for a 20-year-old man who has Hodgkin's lymphoma in the abdominal and pelvic regions. The client is scheduled for radiation therapy and states, " I want to have children someday, and this procedure will destroy my chances." How does the nurse respond? a." Adoption is always an option." b. " Infertility is not seen with this type of radiation therapy." c." Sperm production will be permanently disrupted." d." You have the option to store sperm in a sperm bank."
ANS: D Permanent sterility can occur in male clients receiving radiation in the abdominal and pelvic regions. The client should be informed of this side effect and given the option to store sperm in a sperm bank before treatment. The other options do not appropriately address the client's concerns
17. The nurse is preparing a client with leukemia for a peripheral stem cell transfusion. Which information does the nurse provide the client? a." Nausea and vomiting are common after the transfusion." b. " The transfusion will take about 6 hours." c." You may have numbness in your fingers and toes." d." Your urine may be red for a short time."
ANS: D Red urine can occur as a result of red blood cell breakage within infused stem cells. The cells are transfused during the time frame of an ordinary blood transfusion, numbness and tingling may have been seen during pheresis (not transfusion), and nausea and vomiting may occur during administration of chemotherapy before the stem cell transfusion.
10. The nurse assesses a client who has received thrombolytic therapy after having a myocardial infarction. Which clinical manifestation indicates to the nurse that reperfusion has been successful? a.ST-segment depressionb. Cessation of diaphoresisc.Sudden onset of pleuritic chest paind.Onset of ventricular dysrhythmias
ANS: D The nurse monitors for the following indications of clot lysis and artery reperfusion: cessation of chest pain, sudden onset of ventricular dysrhythmias, resolution of ST-segment depression, and a peak of markers of myocardial damage at 12 hours. Pleuritic chest pain would not occur. ST-segment depression should not occur owing to reperfusion. The client may become less diaphoretic as he or she stabilizes, but this is not a classic sign of reperfusion
9. The nurse is providing care for a client admitted to the hospital with reports of chest pain. After receiving a total of three nitroglycerin sublingual tablets, the client states, " The pain has not gotten any better." What does the nurse do next? a.Place the client in a semi-Fowler's position.b. Administer intravenous nitroglycerin.c.Begin supplemental oxygen at 2 L/min.d.Notify the health care provider.
ANS: D When a client experiences chest discomfort unrelieved by nitroglycerin, the client may be experiencing a myocardial infarction. The provider should be notified and the client prepared for transfer to a unit prepared to provide specialized cardiac care.
12. The nurse is caring for a client who has an elevated white blood cell count. Which intervention does the nurse implement for this client? a.Administer the prescribed Tylenol.b. Hold the client's prescribed steroids.c.Assess the client's respiratory rate.d.Obtain the client's temperature.
ANS: D White blood cells provide immunity and protect against invasion and infection. An elevated white blood cell count could indicate an infectious process, which could cause an elevation in body temperature. Tylenol would treat a fever but not the elevated white blood cell count. Steroids place the client at higher risk for infection but should not be stopped suddenly. The respiratory rate does not need to be assessed in this client.